LSAT and Law School Admissions Forum

Get expert LSAT preparation and law school admissions advice from PowerScore Test Preparation.

 DlarehAtsok
  • Posts: 50
  • Joined: Nov 18, 2015
|
#26893
"Most antidepressant drugs cause weight gain. . .

I get why (C) is correct, but I am not sure whether I understand the book's reasoning behind it. First of all, in the answer key it is claimed that this is a must be true question, but I see it as a most strongly supported question, and that difference is key. Answer (C) states "At least some patients taking antidepressant drugs gain weight as a result of taking them". This does not pass the "Fact Test" since nowhere in the stimulus is mentioned that at least some patients are treated with a drug that cause weight gaining. It may be that all patients buy those antidepressant drugs that do not cause weight gain. On the other hand, on page 272, the book quotes wrongly answer (C) by stating "Some individuals taking antidepressants that cause weight gain will gain weight ..." In fact, answer (C) is correct because it is "most strongly supported" by the facts. This is important, because answers (A) and (B) are eliminated on wrongful ground as exaggerated answers because of the "most" (for the same reason we can eliminate answer (C)). (A) and (B) are incorrect because they paraphrase "suggestions" and nowhere in the stimulus is claimed that weight issues are more important than depression issues (this is also mentioned in the second reason behind the incorrectness of choice (A)).
User avatar
 Jonathan Evans
PowerScore Staff
  • PowerScore Staff
  • Posts: 726
  • Joined: Jun 09, 2016
|
#26903
Hi, Dlareh, good question, and I'm impressed by your excellent dissection of the components of this question. First, this is in fact a "Must be True" question. For the purposes of categorizing questions to enable students to recognize patterns among question types and know what to do to get to the credited response, questions that may be worded differently still fit within the parameters of the same question type. In this case, any question that instructs you to take information from the stimulus to prove or provide evidence for an answer choice is a "Must Be True" question. The same approach works whether it reads, "The information above most strongly supports which of the following," or if it reads, "If the statements above are true, which of the following statements must also be true."

Now with that established, you are correct to note a slight difference of degree between "most strongly supports" and "must be true." The Must Be True questions that include "must be true" in the question stem will be almost incontrovertible with even the most devils-advocate sun-ain't-gonna-shine-anymore suppositions. The "strongly supports" questions might not withstand that level of scrutiny. However, they will be directly and explicitly supported by information in the stimulus and pass the Fact Test. Answer choice "C" does pass the Fact Test. The only way to know that most antidepressant drugs cause weight gain is if such weight gain has been observed more than once among patients taking them. According to the stimulus, such weight gain might be mitigated by dieting but is unlikely to be eliminated entirely. These statements provide ample evidence to know definitively: "At least some patients taking antidepressant drugs gain weight as a result of taking them." Answer choice C with its weak language and direct evidence is a classic credited response for a Must Be True question, irrespective of the particularities of the wording of the question stem.

The answer explanation does not misquote the answer choice but rather elaborates upon the logical connections that lead (C) to be the credited response. As noted above, the fact that we know that "most antidepressant drugs cause weight gain" along with the fact that "some weight gain is unlikely to be preventable" ipso facto gives us proof that "some patients taking antidepressant drugs gain weight as a result of taking them!" There were at least some patients who took these drugs and gained weight.

Remember also the directions at the beginning of the Logical Reasoning sections: "You should not make assumptions that are by commonsense standards implausible." Do not talk yourself out of choosing a credited response by introducing esoteric or far fetched counterfactual assumptions.

You are correct to note that answer choices A & B introduce wholly unsupported opinions. I strongly commend you for your astute observations and deep engagement with the material. You should keep doing exactly what you are doing to prepare.

Get the most out of your LSAT Prep Plus subscription.

Analyze and track your performance with our Testing and Analytics Package.